Đến nội dung

Visitor nội dung

Có 63 mục bởi Visitor (Tìm giới hạn từ 30-05-2020)



Sắp theo                Sắp xếp  

#619288 CMR $x_{n}\equiv x_{n-1}$ với mọi $n...

Đã gửi bởi Visitor on 09-03-2016 - 09:57 trong Số học

Đồng dư module nào thế bạn?



#619275 $[n\sqrt{2}]+[n\sqrt{2}]$ là một số lẻ

Đã gửi bởi Visitor on 09-03-2016 - 00:28 trong Số học

 

2.Chứng minh rằng tồn tại vô số số $n$ nguyên dương sao cho $[n\sqrt{2}]+[n\sqrt{3}]$ là một số lẻ 

+Ta thấy $[(n+1)\sqrt{2}]-[n\sqrt{2}]=1or2$ và $[(n+1)\sqrt{3}]-[n\sqrt{3}]=1or2$

+Giả sử đcm là sai thì sẽ tồn tại $N$ đủ lớn để từ đó trở đi thì $[n\sqrt{2}]+[n\sqrt{3}]$ là chẵn

suy ra là $[(n+1)\sqrt{3}]+[(n+1)\sqrt{2}]-[n\sqrt{3}]-[n\sqrt{2}]$ là chẵn

 

mà $[(n+1)\sqrt{3}]+[(n+1)\sqrt{2}]-[n\sqrt{3}]-[n\sqrt{2}]=2or3or4$ nên $[(n+1)\sqrt{3}]+[(n+1)\sqrt{2}]-[n\sqrt{3}]-[n\sqrt{2}]=2or4$

 

+Do đó $[(n+1)\sqrt{2}]-[n\sqrt{2}]=[(n+1)\sqrt{3}]-[n\sqrt{3}]=1or2$

 

tt :      $[(n+2)\sqrt{2}]-[(n+1)\sqrt{2}]=[(n+2)\sqrt{3}]-[(n+1)\sqrt{3}]=1or2$

...

          $[(n+c)\sqrt{2}]-[(n+c-1)\sqrt{2}]=[(n+c)\sqrt{3}]-[(n+c-1)\sqrt{3}]=1or2$

 

+Cộng lại ta được: $[(n+c)\sqrt{2}]-[n\sqrt{2}]=[(n+c)\sqrt{3}]-[n\sqrt{3}]$

 

+Mặt khác $[(n+c)\sqrt{2}]-[n\sqrt{2}]< (n+c)\sqrt{2}-n\sqrt{2}+1= c\sqrt{2}+1$

 

và     $[(n+c)\sqrt{3}]-[n\sqrt{3}]> (n+c)\sqrt{3}-1-n\sqrt{3}= c\sqrt{3}-1$ 

 

nên là $c\sqrt{2}+1> c\sqrt{3}-1\Rightarrow 2> c(\sqrt{3}-\sqrt{2})$

 

+chọn $c$ đủ lớn thì vô lí và ta có $đpcm$

p/s: hơi xấu =.=




#618560 Tìm $(x, y) \in \mathbb{Z}^{2}$:...

Đã gửi bởi Visitor on 05-03-2016 - 20:04 trong Số học

Lời giải của bạn rất hay nhưng có chỗ không đúng. Lý do: Ký hiệu Legendre chỉ dùng cho số nguyên tố lẻ thôi
Tuy nhiên vẫn có cách khắc phục: $y^{2} + 2y + 4 \vdots 2 \iff y \vdots 2 \iff x \vdots 2$. Mặt khác để ý $\text{VT} \equiv 2 \pmod{4}$ và $y^{3} \equiv 0\pmod{4}$. Vô lí, do đó $y^{2} + 2y + 4$ lẻ, áp dụng bài toán của bạn là xong.
P.s: mình có 1 ý tưởng khác xấu xí tí. :-). À Visitor này, mình hỏi riêng bạn chuyện này được không nhỉ?

lâu ko dùng Legendre quên mất :-) 
P.s: ừ,bạn ib đi :-)




#618439 Chứng minh rằng mọi số nguyên lớn hơn 11 là tổng của hai hợp số.

Đã gửi bởi Visitor on 05-03-2016 - 01:25 trong Số học

 

2. Chứng minh rằng nếu  $f(x)=a_{n}x^N+a_{n-1}x^{n-1}+...+a_{1}x+a_{0}$ 

    là đa thức với hệ số nguyên thì tồn tại $y$ sao cho $f(y)$ là hợp số.

P/s:   Help me!! 

Giả sử $a_n> 0$ khi đó $limf(x)=+vc$. Vì để bài ko nói $y$ nguyên hay ko nên ta thấy $f(x)$ liên tục nên luôn tồn tại $y$ để $f(y)$ là hợp số =))

chơi lầy =))




#618438 CMR $\text{gcd}(2^{n} - 1; 3^{n} + 2)...

Đã gửi bởi Visitor on 05-03-2016 - 00:44 trong Số học

Cho $n$ là số nguyên dương. CMR $\text{gcd}(2^{n} - 1; 3^{n} + 2) = 1$

Bài này quá khó, bối rối quá. Ego post lời giải được ko.




#618437 Tìm $(x, y) \in \mathbb{Z}^{2}$:...

Đã gửi bởi Visitor on 05-03-2016 - 00:39 trong Số học

Tìm $(x, y) \in \mathbb{Z}^{2}$:
$$9x^{2} + 6x + 10 = y^{3}$$

$pt\Leftrightarrow (3x+1)^2+1=(y-2)(y^2+2y+4)$

Gọi $p$ là ước nguyên tố bất kì của $y^2+2y+4$ thì $p|(3x+1)^2+1$

$\Rightarrow (\frac{-3}{p})= 1,(\frac{-1}{p})=1\Rightarrow \Rightarrow  (\frac{3}{p})= 1$

suy ra $p\equiv \pm 1(mod12)\Rightarrow y^2+2y+4\equiv \pm 1(mod12)$ 

vô lí vì $ y^2+2y+4\not\equiv \pm 1(mod12)$

PT vô nghiệm 




#617272 $\gcd (n,1)+ \gcd (n,2)+ \cdots + \gcd (n,n) = 3n-3....

Đã gửi bởi Visitor on 27-02-2016 - 21:28 trong Số học

Hình như không đúng. Mình chứng minh được số cần tìm là chẵn và square-free, bị đứng một chỗ. Và bạn thử lại xem. $n = 2p$ luôn thỏa mãn với $p$ là số nguyên tố lẻ :-) ($n = 10, n = 14$ vẫn thỏa mãn)

thank Ego, nãy mình tính sai tổng xích ma ,đã edit  :)




#617245 Chứng minh rằng $\frac{a_{n}}{n}...

Đã gửi bởi Visitor on 27-02-2016 - 20:03 trong Dãy số - Giới hạn

Cho dãy số (an) xác định bởi a=0, a1=1, và an+2 = 2an+1 +an với mọi số tự nhiên n. Chứng minh rằng $\frac{a_{n}}{n}$ khi viết dưới dạng tối giản thì cả tử và mẫu đều là các số lẻ với $n\geq 1$                

Bài này mình đoán là tìm cttq của dãy, rồi đi chứng minh qui nạp tử số mũ của $2$ ở tử số là $n$, hay cái gì đó. mẫu là $2^n$.

định làm hẳn hoi nhưng lâu lâu quên mất cách tìm cttq -.-




#617244 $\gcd (n,1)+ \gcd (n,2)+ \cdots + \gcd (n,n) = 3n-3....

Đã gửi bởi Visitor on 27-02-2016 - 19:57 trong Số học

(Bài toán cũ) Tim số nguyên dương $n$ sao cho $$\gcd (n,1)+ \gcd (n,2)+ \cdots + \gcd (n,n) = 3n-3.$$

Gọi $d=(k,n)$ thì $(\frac{k}{d},\frac{n}{d})=1$ nên ta có thể viết lại vế trái dưới dạng $\sum_{d|n}d\phi (\frac{n}{d})$

Ta áp dụng công thức siêu kinh điển : Với $f,g$ là hai hàm nhân tính thì $\sum_{d|n}f(d)g(\frac{n}{d})= \prod _{p|n}(\sum_{k=0}^{v_p(n)}f(p^k)g(p^{v_p(n)-k}))$

Ở đây ta chọn $f(x)=x,g(x)=\phi (x)$ thì $\sum_{d|n}d\phi (\frac{n}{d})= \prod _{p|n}(\sum_{k=0}^{v_p(n)}p^k\phi (p^{v_p(n)-k}))=\prod _{p|n}(\sum_{k=0}^{v_p(n)-1}p^k.p^{v_p(n)-k-1}(p-1)+p^{v_p(n)})$

$ = \prod _{p|n}(\sum_{k=0}^{v_p(n)-1}p^{v_p(n)-1}(p-1)+p^{v_p(n)})= \prod _{p|n}(p^{v_p(n)-1}(v_p(n)(p-1)+1))$                                            

Do đó, nếu tồn tại $p|n$ mà $v_p(n)\geq 2$ thì $p^{v_p(n)-1}|3\Rightarrow p=3$ và  $v_3(n)= 2$ 

suy ra mọi ước nguyên tố $p$ khác $3$ thì $v_p(n)=1$

Vậy ra rút gọn được tiếp vế trái bằng $  \prod _{p|n}(p^{v_p(n)-1}(p+1(p-1)))= 3(3+2.2)\prod _{p|n,p\neq 3}p^{v_p(n)-1}(2p-1)=3n-3$

Nếu $n$ lẻ thì vô lí vì vế trái lẻ còn vế phải chẵn, suy ra $n$ chẵn. Lại rút gọn tiếp:

$3n-3= (2.2-1).3.(3+2.2)\prod_{p|n,p\neq 2,3} p^{v_p(n)-1}(2p-1)\vdots 9$ vô lí

$\Rightarrow v_p(n)=1$ với mọi $p|n$

Tóm lại $3n-3=\prod _{p|n}p^{v_p(n)-1}(2p-1)$                                                         $(1)$

 

Giả sử $n=2p_1p_2...p_k$ , $p_i$ lẻ thì từ  $(1)$ suy ra $3(2p_1-1)(2p_2-1)...(2p_k-1)=6p_1p_2...p_k-3$.

Đánh giá bđt đơn giản ta được $k=1$ , dẫn đến $3(2p_1-1)=6p_1-3$

Thỏa mãn với mọi $p$

Vậy $n=2p$




#617233 Chứng minh rằng $B$ có thể chọn số $k$ khéo léo để chỉ mộ...

Đã gửi bởi Visitor on 27-02-2016 - 19:17 trong Số học

Hôm trước mình làm sai, mình làm lại như sau:

Giả sử các phần tử của $S$ là $a_1,a_2,...a_n$ và tất cả các ước nguyên tố của các $a_i$ là $p_1,p_2,...p_k$

và $a_1=p_1^{m_{11}}p_2^{m_{12}}...p_k^{m_{1k}}$, ... $a_i=p_1^{m_{i1}}p_2^{m_{i2}}...p_k^{m_{ik}}$

Ta sẽ chọn $K=p_1^{n_1}p_2^{n_2}...p_k^{n_k}$ với các $n_i$ thíc hợp

Khi đó $d(Ka_i)=(n_1+m_{i1}+1)(n_2+m_{i2}+1)...(n_k+m_{ik}+1)$

kiểu bài này là dùng $CRT$ luôn , gọi ra tập $P$ gồm $n.k$ số nguyên tố $q_{i1},q_{i2},...q_{ik}$ với $i$ chạy từ $1$ đến $n$

Với mỗi $j$ từ $1$ đến $k$ ta chọn $n_j$ sao cho $n_j+m_{ij}+1$ chia hết cho $q_{ij}$ , $i$ chạy từ $1$ đến $n$, và ko chia hết cho các số nguyên tố còn lại.

Vì chỉ có hữu hạn các tổng $n_j+m_{ij}+1$ nên ta có thể chọn các $q_{ij}$ đủ lớn để ko có $2$ số nào cùng chia hết cho $1$ số nguyên tố thuộc $P$

Khi đó mỗi tích  $q_{i1}q_{i2}...q_{ik}$ chỉ chia hết duy nhất $d(Ka_i)$. Vì thế $B$ chỉ cần một lượt chơi để tìm ra số $a_i$ .

 $đpcm$




#617215 CMR tồn tại số $x$ sao cho $a_1+x,a_2+x,...,a_n+x$ là số...

Đã gửi bởi Visitor on 27-02-2016 - 18:10 trong Số học

 

 

2/Tìm tất cả các số $n\in \mathbb{Z^+}$ có tính chất sau: Tập hợp $S=\{n;n+1;n+2;....;n+5\}$ có thể chia thành hai tập hợp con $X,Y$ sao cho tích các phần tử của $X$ bằng tích các phần tử của $Y$

cách $2$ cho bài $2$ :))

ta thấy rằng trong $6$ số tự nhiên liên tiếp chỉ nhiều nhất một số chia hết cho $7$.

Nếu trong tập $S$ có $1$ số chia hết cho $7$ thì giả sử số đó thuộc $X$. Khi đó tích các phần tử của $X$ sẽ chia hết cho $7$ còn $Y$ thì ko.Loại

Nếu trong $S$ ko có số nào chia hết cho $7$, thì khi chia tập $S$ cho $7$ sẽ được các số dư là $1,2,3,4,5,6$

Giả sử chia được thành $n(n+1)=(n+2)(n+3)(n+4)(n+5)$ ( cái này vô lí thật nhưng chỉ để tượng trưng thôi)

suy ra $[n(n+1)]^2= n(n+1)(n+2)(n+3)(n+4)(n+5)\equiv  1.2.3.4.5.6=6!\equiv -1$ $(mod 7)$

$\Rightarrow X^2+1\vdots 7$

vô lí vì mọi ước nguyên tố của $X^2+1$ chỉ có dạng $4k+1$

Vậy ko có $n$




#617212 CMR tồn tại số $x$ sao cho $a_1+x,a_2+x,...,a_n+x$ là số...

Đã gửi bởi Visitor on 27-02-2016 - 17:56 trong Số học

 

 

2/Tìm tất cả các số $n\in \mathbb{Z^+}$ có tính chất sau: Tập hợp $S=\{n;n+1;n+2;....;n+5\}$ có thể chia thành hai tập hợp con $X,Y$ sao cho tích các phần tử của $X$ bằng tích các phần tử của $Y$

Thôi đã ghé qua THCS rồi thì làm nốt :3

nếu chia được thì sẽ có một tập chứa $n$ do đó tích chia hết cho $n$ , rồi xét modulo $n$ hai vế , chắc hơi dài :v




#617208 CMR tồn tại số $x$ sao cho $a_1+x,a_2+x,...,a_n+x$ là số...

Đã gửi bởi Visitor on 27-02-2016 - 17:39 trong Số học

1/Cho $n$ số thực $a_1,a_2,...,a_n$ bất kì.Chứng minh rằng tồn tại số $x$ sao cho $a_1+x,a_2+x,...,a_n+x$ là số vô tỉ

 

 

Bài này cấp 2 đúng rồi :))

Phản chứng giả sử ko tồn tại $x$ . Gọi $t$ là một số vô tỉ bất kì

Xét bảng sau:

$t+a_1$           $t+a_2$            $t+a_3$ ...            $t+a_n$

$2t+a_1$         $2t+a_2$          $2t+a_3$ ...          $2t+a_n$

...

$nt+a_1$         $nt+a_2$          $nt+a_3$ ...          $nt+a_n$

$(n+1)t+a_1$   $(n+1)t+a_2$   $(n+1)t+a_3$ ...   $(n+1)t+a_n$

Từ giả thiết phản chứng ta suy ra mỗi hàng sẽ có ít nhất một số hữu tỉ, nên cả bảng sẽ có ít nhất $n+1$ số hữu tỉ.

Lại có $n$ cột nên tồn tại ít nhất hai số cùng cột, giả sử $2$ số hữu tỉ là $pt+a_k$ và $qt+a_k$.

suy ra $(p-q)t$ là hữu tỉ. Vậy vô lí. $đpcm$ 




#616972 chứng minh x thuộc Z

Đã gửi bởi Visitor on 25-02-2016 - 23:55 trong Số học

cho x^3-x thuộc Z,x^4-x thuộc Z chứng minh x thuộc Z

Bài này thật vi diệu :v 

Giả thiết suy ra $\frac{x^4-x}{x^3-x} \in \mathbb{Q}\Leftrightarrow \frac{x^3-1}{x^2-1}\in\mathbb{Q}\Leftrightarrow \frac{x^2+x+1}{x+1}\in \mathbb{Q}\Leftrightarrow \frac{x^2}{x+1}=b\in\mathbb{Q}$

Suy ra $(x^3-x).\frac{x+1}{x^2}= \frac{x^3+x^2-x-1}{x}=c\in\mathbb{Q}$

 đặt $x^3-x=a$ thì $\frac{a+x^2-1}{x}=c\Rightarrow a+x^2-1=cx$

                                                 mà $\frac{x^2}{x+1}= b\Rightarrow x^2=bx+b$

 

Trừ theo vế ta được $a-1=(c-b)x-b$

Dễ thấy $b\neq c$ nên ta sẽ có $x\in\mathbb{Q}$

Ta xét phương trình hệ số nguyên $t^3-t-(x^3-x)=0$ có $x$ là nghiệm hữu tỉ nên nó cũng là nghiệm nguyên.

$đpcm$ 




#616801 Chứng minh n=1

Đã gửi bởi Visitor on 25-02-2016 - 07:38 trong Số học

Cho 3 số nguyên tố p,q,r thỏa mãn $p^{n}+q^{n}=r^{2}$.

Chứng minh n=1

Giả sử $q=2$

Nếu $n$ lẻ thì ta có $p^n+2^n=(p+2)\frac{p^n+2^n}{p+2}= r^2$ mà $r$ nguyên tố suy ra $p+2=\frac{p^n+2^n}{p+2}= r$

Do đó $n=1$ vì nếu $n\geq 3$ thì $\frac{p^n+2^n}{p+2}> p+2$ vô lí.

Nếu $n$ chẵn, $n=2k$ suy ra $(p^k)^2+(2^k)^2=r^2$ .Dễ thấy $p,2,r$ phân biệt nên đây là bộ $Pitago$ nguyên thủy nên tồn tại $m,n$ nguyên tố cùng nhau, khác tính chẵn lẻ sao cho $p^k=2mn, 2^k=m^2-n^2$ hoặc $2^k=2mn, p^k=m^2-n^2$

Vì $p$ nguyên tố lẻ nên cả $2$ trường hợp đều vô lí.

Vậy $n=1$




#616292 Chứng minh rằng $B$ có thể chọn số $k$ khéo léo để chỉ mộ...

Đã gửi bởi Visitor on 21-02-2016 - 20:07 trong Số học

Cho một tập hữu hạn $S$ gồm các số nguyên dương. Cả hai người A và B đều biết trước các phần tử của $S$. Họ thực hiện trò chơi như sau: A lấy một số $m$ bất kì thuộc $S$ (không để B biết); tiếp theo B chọn một số $k$ bất kì (không nhất thiết thuộc $S$); sau đó A tính toán và cho $B$ biết giá trị của $d(km)$ với $d(n)$ là số các ước số dương của $n$ để B đoán ra số $m$; làm như vậy cho đến khi B đoán đúng. Chứng minh rằng $B$ có thể chọn số $k$ khéo léo để chỉ một lượt chơi là B có thể tìm được số như yêu cầu.

p.s. Đây là đề qgia của Ukraina năm nay. Mình có một lời giải nhưng không nghĩ nó hay cho lắm, đăng lên để anh em tham khảo cho ý kiến.




#614893 Tồn tại vô hạn

Đã gửi bởi Visitor on 14-02-2016 - 13:08 trong Số học

Cho một số nguyên dương $a$. Chứng minh rằng với mọi $m$, tồn tại vô hạn số nguyên dương $n$ thỏa mãn số ước số của $na^{n} + 1$ thì chia hết cho $m$.

Chọn $p$ là số nguyên tố lẻ mà $v_p(a^{p-1}-1)=1$

để tránh dài dòng mình dùng $v_p$ để diễn tả cho gọn, chứ ko phải là dùng LTE   :))

1.Ta sẽ chứng minh bằng qui nạp theo $k$ rằng tồn tại vô số $n$ để $v_p(na^{n}+1)=k$             $(*)$

Trước tiên là ta nhận xét rằng nếu $v_p(na^{n}+1)=k$ thỏa mãn thì $v_p(ma^{m}+1)$, với $m=n+tp^k(p-1)$ cũng bằng $k$, thật vậy:

Giả sử $na^{n}+1=p^k.A$ 
Ta có $ma^m+1=(n+tp^k(p-1))a^{n+tp^k(p-1)}=na^n(a^{tp^k(p-1)}-1)-(na^n+1)+tp^k.B$
$=na^n(a^{tp^k(p-1)}-1)+p^k(tB-A)$

Do $(A,p)=(B,p)=1$ nên có thể chọn $t$, $(t,p)=1$ để mà $tB-A\not \vdots p$. Mà $v_p(a^{tp^k(p-1)}-1)=k+1$ nên rõ ràng $v_p(ma^m+1)=k$

Vậy $NX$ ở trên là đúng, từ đây suy ra nếu tồn tại $n$ thì sẽ có vô hạn.Do đó ta chỉ cần chứng minh tồn tại

Bắt đầu qui nạp.

+Với $k=1$ chọn $n=p^2-1$ thì $na^n+1=(p^2-1)a^{p^2-1}+1=p^2a^{p^2-1}-(a^{p^2-1}-1)$

do cách chọn thì ta có $v_p(a^{p^2-1}-1)=1$ nên suy ra $v_p(na^n+1)=1$, thỏa mãn

+Giả sử qui nạp đúng đến $k$ , ta cm vs $k+1$ 

Chọn $m=n+tp^{k+1}(p-1)$ thì ta sẽ có $v_p(ma^m+1)=k+1$, chứng minh giống y hệt như chứng minh nhận xét ở trên.

 

2. từ $(*)$ ta suy ra ngay đpcm




#612424 Chứng minh rằng dãy $(p_n)$ bị chặn trên.

Đã gửi bởi Visitor on 02-02-2016 - 14:16 trong Số học

Cho dãy $(p_n)$ là dãy các số nguyên tố thỏa mãn với mọi $n \geqslant 3$ thì ta có $p_n$ là ước nguyên tố lớn nhất của $p_{n-1}+p_{n-2}+2000.$.

Chứng minh rằng dãy $(p_n)$ bị chặn trên. 

Nếu cả $2$ số $p_{n-1}, p_{n-2}$ đều lẻ thì ta có $p_{n-1}+p_{n-2}+2000$ là chẵn. Và do tính lớn nhất của $p_n$ nên suy ra $p_n \leq \frac{p_{n-1}+ p_{n-2}+2000}{2} \leq max(p_{n-1}, p_{n-2}) +1000$

Nếu trong $2$ số $p_{n-1}, p_{n-2}$ có $1$ số bằng $2$ thì $p_n  \leq max(p_{n-1}, p_{n-2}) +2+2000\leq max(p_{n-1}, p_{n-2})+2002$

Từ $2$ trường hợp trên ta rút ra $p_n  \leq max(p_{n-1}, p_{n-2})+2002$ với mọi $n$

Do đó nếu đặt dãy $q_n=max(p_{n},p_{n-1})$ thì ta có $q_n \leq q_{n-1}+2002$ với mọi $n$ . ta đi chứng minh dãy $q_n$ bị chặn là xong.

Ta sẽ chứng minh dãy bị chặn bởi số $M$ thỏa mãn: $ q_1 \leq M$ và tất cả các số $M+1,M+2,...M+2002$ đều là hợp số ( số $M$ này luôn chọn được theo định lí $CRT$ ) , thật vậy : do $q_1< M\Rightarrow q_2\leq q_1+2002<M+2002$ . Lại có là từ $M+1$ đến $M+2002$ ko có số nào là số nguyên tố. Mà $q_2$ là số nguyên tố nên nhất định là $q_2 \leq M$

Cứ như vậy ta sẽ chứng minh được $q_n \leq M$ với mọi $n$




#612379 $F_{10^{k}}$ $(k{\geq}1)$ luôn tận cùng bằng 5

Đã gửi bởi Visitor on 01-02-2016 - 23:45 trong Số học

vậy bạn có cm được bài 2 không? Hoặc co chứng minh được bổ đề đó không? Nhớ là tự làm nhé! Hoặc nếu theo ý kiến của bạn thì bạn có thể chứng minh bài 2 sai không? nếu làm không được thì đừng có nói nhé. Khoa học cần bằng chứng chứ không cần lời nói.

Mình ko đủ khả năng để chứng minh bài toán của bạn đúng hay sai. NHƯNG mình xin kể $1$ câu chuyện như sau :)

Có một định đề mang tên nhà toán học $Bertrand$ là : Với mọi $n>1$ thì giữa $n$ và $2n$ luôn có 1 số nguyên tố. Định đề này sau đó đã được $Chebyshev$ chứng minh.Cm cho định lí này rất khó. Cách sơ cấp nhất mà theo mình nhớ là được đưa lên báo TH và TT năm kia , trong đó dùng tới gần cả chục bổ đề nhỏ.

Như thế là đủ hiểu cm khó như nào.

Quay trở lại bài toán của bạn. Giả sử rằng nó đúng. như ở trên mình có viết là với $n$ đủ lớn thì $(n+1)^2<2n^2$ , tức là với $k$ đủ lớn ( tầm 25 trở đi) thì trong đoạn $[k,2k]$ sẽ chứa đoạn $[n^2,(n+1)^2]$ với $n$ nào đó. Mà theo bài toán của bạn thì đoạn này sẽ chứa ít nhất $1$ số nguyên tố. Do đó định lí $Bertrand$ đã được chứng minh.
Vậy bạn có thể đưa ra chứng minh cho bài toán $2$ để chúng ta có $1$ cách chứng minh rất mới cho định lí $Bertrand$  được ko :)

 

p/s : bạn ạ. diễn đàn là nơi trao đổi thảo luận về các bài toán :) dù mình ko làm được nhưng mình đưa ra ý kiến để thảo luận là đúng với ý chí của diễn đàn rồi. cách nói của bạn như kiểu đây là nơi chỉ để hỏi bài hoặc là đưa bài ra để thách thức vậy :)




#612375 $F_{10^{k}}$ $(k{\geq}1)$ luôn tận cùng bằng 5

Đã gửi bởi Visitor on 01-02-2016 - 23:16 trong Số học

thử trường hợp với $k=5$ nhé! ta đặt vật và quả cân $3^{0}$ và quả cân $3^{1}$ lên 1 đĩa đĩa kia đặt quả cân $3^{2}$ là được rồi :)

Ra là mình hiểu nhầm đề bài :) Vậy thì bài này mình quy nạp xem sao :) Giả sử bài toán đúng tới $k-1$ ta sẽ chưng sminh nó đúng với $k$

Bài toán đúng với $t$ thì ta kí hiệu $t-True$ :) . Kí hiệu tập các quả cân làm cho $t-True$ là $A_t$

Nếu $k=3t$ thì theo gtqn $t-True$ , lấy $A_k=3A_t$ là xong , từ đây ta có nhận xét là nếu $k-True$ và $k$ chia hết cho $3$ thì tất cả các quả cân trong $A_k$ đều nặng hơn $1$

Nếu $k=3t+1$ , lấy $A_k$ bằng cách thêm vào $A_{3t}$ quả cân $1$.

Nếu $k=3t+2$ thì ta làm lùi từ $h=3(t+1)$ bằng cách thêm vào đĩa chứa $h$ quả cân $1$ . Do $1$ chưa xuất hiện trong $A_h$ ( suy ra từ nhận xét ở trên)  .

Vậy ta có đpcm




#611980 Tìm GTNN của: P=$\frac{a^{3}+b^{3}+c^...

Đã gửi bởi Visitor on 31-01-2016 - 18:12 trong Bất đẳng thức - Cực trị

Cho $a,b,c> 0$.Tìm GTNN của: P=$\frac{a^{3}+b^{3}+c^{3}}{abc}+\frac{(a+b+c)^{2}}{a^{2}+b^{2}+c^{2}}$.

@@ 




#611979 Tìm tất cả các số nguyên tố $p$ sao cho $2^n-2$ không chi...

Đã gửi bởi Visitor on 31-01-2016 - 18:05 trong Số học

Với $p$ là số nguyên tố, đặt $n=\frac{2^{2p}-1}{3}$. Tìm tất cả các số nguyên tố $p$ sao cho $2^n-2$ không chia hết cho $n$

Nếu $p\neq 2,3$ thì $n-1=\frac{4^p-4}{3}=\frac{4(4^{p-1}-1)}{3}\vdots 2p\Rightarrow 2^n-2=2(2^{n-1}-1)\vdots (2^{2p}-1)\vdots n$

Vậy $p$ có thể bằng $2$ hoặc $3$ :v




#611857 $F_{10^{k}}$ $(k{\geq}1)$ luôn tận cùng bằng 5

Đã gửi bởi Visitor on 30-01-2016 - 22:59 trong Số học

Bài 1 (QVLuom) Cho vô hạn quả cân dạng $3^{i}$ với i=0,1,2,... mỗi loại 1 quả. Chứng minh rằng tồn tại một cách đặt duy nhất các quả cân lên chiếc cân đĩa sao cho với một vật có khối lượng k nguyên dương bất kì để cân thăng bằng.

Bài 2 (QVLuom) Chứng minh rằng tồn tại ít nhất 1 số nguyên tố nằm giữa 2 số chính phương liên tiếp $n^{2}$ và $(n+1)^{2}$ với $n{\geq}1$.

Bài 3 (Thelongduong) Chứng minh rằng số $F_{10^{k}}$ $(k{\geq}1)$ luôn tận cùng bằng 5 với $F_{n}$  là số Fibonacci thứ n

Bài 1 : nếu $k$ mà chia $3$ dư $2$ thì đặt quả cân kiểu gì :))
Bài 2 : ta có với $n$ đủ lớn thì ta có $(n+1)^2 < 2n^2 $ . Vậy là :)) nếu bài toán của tác giả là đúng thì nó sẽ mạnh hơn cả bổ đề $Bertrand$ :)) Điều này là ko thể 




#611275 $Cho k \epsilon \mathbb{N} k>1 .Cmr$ ton ta...

Đã gửi bởi Visitor on 27-01-2016 - 13:55 trong Số học

$Cho  k  \epsilon \mathbb{N} , k>1 .Cmr$ ton tai huu han x thoa man:

    $\sigma (x)-x=k$

Cách khác đơn giản hơn nhé :)

Nếu $x$ là số nguyên tố thì vô lí. Xét $x$ là hợp số. Gọi $d$ là một ước của $x$ thì hiển nhiên $\frac{x}{d}$ cũng là ước của $x$ 

Ta chọn $d$ là ước nguyên tố của $x$ mà ko vượt quá $\sqrt{x}$ ( luôn tồn tại )

Suy ra $\sigma (x)\geq 1+x+\frac{x}{d}= 1+x+\frac{x}{d}+d-d\geq1+x+2\sqrt{x}-d\geq 1+x+\sqrt{x}$

kéo theo $\sigma (x)-x\geq 1+\sqrt{x}$.  Do đó sẽ có hữu hạn $x$ thỏa đề.




#611271 $x^n+y^n+z^n+t^n=u^{n+1}$

Đã gửi bởi Visitor on 27-01-2016 - 13:29 trong Số học

1. Chứng minh rằng phương trình sau có vô số nghiệm nguyên:

$$x^5+y^5+z^5=x^4+y^4+z^4$$

 

Đầu tiên chọn $y=-x$ cho gọn thành $z^5=2x^4+z^4$

chia cả 2 vế cho $z^4$ được $z=2(\frac{x}{z})^4+1$. Dễ quá rồi :)) , chọn ngay $z=2k^4 +1 $ và $x=kz$ là vô số nghiệm luôn :))